Ejercicio con proceso de Poisson y exponencial

Tengo este problema que no logro comprender. Saludos experto y muchas gracias de antemano.

El número de defectos en los rollos de tela producidos en cierta máquina, es un proceso Poisson {N(t)} con tasa lambda defectos por rollo. La probabilidad de que en los primeros 2 rollos de tela haya un defecto, es 1.6e^-1.6

a) ¿Cuál es el número esperado de rollos entre dos defectos consecutivos?

b) ¿Cuál es la esperanza del número de rollo en el que aparecerá el tercer defecto?

1 Respuesta

Respuesta
1

a)

Lambda es el número esperado de defectos por rollo. Aquí t será el número de rollos.

En t=2 la probabilidad de 1 defecto es

$$\begin{align}&P(N(2)=1) = \frac{e^{-2\lambda} ·(2\lambda)^1 }{ 1!} = 2\lambda·e^{-2\lambda}\\ &\\ &\text{lo cual debe ser igual a la probabilidad que nos dan}\\ &\\ &2\lambda e^{-2\lambda}=1.6e^{-1.6}\\ &\end{align}$$

Una ecuación de esta clase exponencial por polinómica solo se puede resolver en situaciones preparadas, tal como en esta que se puede ver que lambda=0.8
Luego el número esperado de defectos por rollo es 0.8

El número esperado de rollos entre dos defectos consecutivos es el inverso del número esperado de defectos por rollo

1/0.8 = 1.25

b) La probabilidad de que sea el rollo primero es

1 - P[N(1)=0] - P[N(1)=1] - P[N(1) = 2]

La probabilidad de que sea en el rollo segundo es

P[N(1)=2]·(1-P[N(1)=0])

La probabilidad de que sea en el rollo tercero es

P[N(2)=2]·(1-P[N(1)=0])

En el cuarto será:

P[N(3)=2]·(1-P[N(1)=0])

....

en el rollo enésimo

P[N(n-1)=2]·(1-P[N(1)=0])

Luego la esperanza será:

$$\begin{align}&E(\text{rollo con el defecto tercero})=\\ & \\ & 1-e^{-0.8}-0.8e^{0.8}-\frac{0.8^2}{2}e^{-0.8} +\\ & \\ & \sum_{k=2}^{\infty}k·\frac{e^{-0.8k}·(0.8k)^2}{2}·(1-e^{-0.8})=\\ & \\ & 1-2.12e^{-0.8}+0.32(1-e^{-0.8})\sum_{k=2}^{\infty}k^3·e^{-0.8k}\end{align}$$

Por supuesto que la parte del sumatorio no lo sé deducir, no sé si alguien sabrá, resuelto con WolframAlpha

http://www.wolframalpha.com/input/?i=sum+of+n^3%C2%B7e^{-0.8n}+from+n%3D2+to+infinity

14.2062

A lo mejor podríamos haberlo hecho más exacto, simplemente con una calculadora Casio Fx-991-ES que tiene sumatorios, haciendo 120 sumas y el rato que le cuesta obtenemos

14.20624047

Y con ello el resultado es:

$$\begin{align}&1-2.12e^{-0.8}+0.32(1-e^{-0.8})14.20624047=\\ &\\ &2.550771446\end{align}$$

Y ese es el resultado, habrá que tener fe en que esté bien, yo creo que sí.  Aunque pienso que habrá alguna teoria que haga el cálculo de otra forma exacta, pero yo no la conozco.

Añade tu respuesta

Haz clic para o

Más respuestas relacionadas